John gets paid $76 a day at his new job printing t-shirts. He also earns $2.50 for every t-shirt sold. Create
and equation to describe how much money John makes in a day. Write the equation for this problem in
function notation. Then interpret how much money John would makes in one day after selling 32 t-shirts.
Make sure to define your variables and show all of your work.I swear who ever answers this I will Give them brainlist please help

Answers

Answer 1

Answer:

y=76+2.5x where y is the amount he earns in a day and x is the number of tshirts sold

y=76+2.5(32)

y=76+80

y=156

he earns 156 dollars if he sold 32 tshirts

Step-by-step explanation:


Related Questions

1. In Tagaytay Picnic Groove, a horse ride costs P 120.00 per person for the first 30
minutes and P 50.00 for every additional 10 minutes. If you spent P 270.00 for a
horse ride, for how long did you ride?​

Answers

Please mark Brainliest

Answer:
1 Hour

Explanation:


•A horse ride costs P120.00 per person for the
first 30 minutes.
•P50.00 for every additional 10 minutes.
I Spend P270.00, I Subtracted P120.00 For the
first 30 minutes.
270 - 120 = 150
150/50 = 3
If I Spend P50.00 for every additional 10 minutes
Then:
3 * 10 = 30
P120.00 For the first 30 minutes.
30 minutes + 30 minutes = 60 min = 1 Hour

Which expressions are equivalent to One-half + 3 c + StartFraction 4 Over 5 EndFractionCheck all that apply.

Answers

Answer:

(13+30c)/10

Step-by-step explanation:

Given the expression;

1/2 + 3c + 4/5

Find the LCM

= 5(1)+10(3c)+2(4)/10

=  (5+30c+8)/10

= (13+30c)/10

Hence the required sum is (13+30c)/10

A salesman is paid $450 a week, plus a commission of 35 cents for every item that he sells. Write a linear function where the paycheck, P, of the salesman is a function of the number, N, of items he sells. If the salesman only has 1400 items available to sell each week, find the domain and range of the function.

Answers

Answer:

Step-by-step explanation:

1400×0.35

490+450

940

Answer:

The domain is thus [0, 1400].

The range is thus [$450, $940]

Step-by-step explanation:

A suitable function is P(N) = ($450) + ($0.35 / item)*N, where $450 is the base pay per week and N is the number of items sold.

In this case the salesman has only 1400 items available to sell each week.  Therefore N begins at 0 (no items to sell) and ends at 1400 (the max number of items available to sell).  The domain is thus [0, 1400].

The smallest amount the salesman could receive would be when he has not sold any items:  $450.  

The max amount is then $450 + ($0.35/item)(1400 items), or

                                         $450 + $490, or

                                          $940

The range is thus [$450, $940]

helppppppppppppppppppppppp

Answers

Answer:

gotta be  answer A

Step-by-step explanation:

you can search up f(x) = square root of x

according to a survey, the population of a city doubled in 12 years.
The annual rate of increase of the population of this city is approximately _____. The population will grow to three times its current size in approximately ______.

First box of answers: 2.50, 5.78, 12.0, 50.0

Second box of answers: 18, 19, 23, 24.

Answers

Answer:

5.78

19

Step-by-step explanation:

Let original population be, P = x

Growth in 12 years, A = 2x

Rate be = r

Time = 12years

Find the rate :

   [tex]A = P(1 + \frac{r}{100})^t[/tex]

 [tex]2x = x(1 + \frac{r}{100})^{12}\\\\\frac{2x}{x} =(1 + \frac{r}{100})^{12}\\\\2 = (1 + \frac{r}{100})^{12}\\\\ \sqrt[12]{2} = (1 + \frac{r}{100})\\\\\sqrt[12]{2} - 1 = \frac{r}{100}\\\\2^{0.08} - 1 = \frac{r}{100}\\\\1.057 - 1 = \frac{r}{100}\\\\0.057 \times 100 = r\\\\r = 5.7 \%[/tex]

The annual rate of increase of the population of this city is approximately 5.78.

Find  time in which the population becomes 3 times.

That is A = 3x

P = x

R= 5.78%

               [tex]A = P( 1 + \frac{r}{100})^t\\\\3x = x ( 1 + \frac{5.78}{100})^t\\\\3 = (1.0578)^t\\\\log \ 3 = t \times log \ 1.0578 \\\\t = \frac{log \ 3}{ log \ 1.0578 }\\\\t = 19.55[/tex]

The population will grow to three times its current size in approximately 19years .

     

5.78% ,19 years are the answers.

2=(1+r)^12

r=(2)^(1÷12)−1

R=0.0578*100=5.78%

3=(1+0.0595)^t

t=log(3)÷log(1.0595)

t=19 years

What is an exponential growth model?

Exponential growth and exponential decay are two of the most common uses of exponential functions. Systems with exponential growth follow a model of the form y = y0ekt. In exponential growth, the growth rate is proportional to the amount present. In other words, for y'= ky

exponential function, multiply a by x to produce y. The exponential graph looks like a curve that starts with a very flat slope and becomes steep over time.

The exponential model, like the sphere model, starts at the origin and operates linearly near it. However, the increasing slope of the curve is less than the slope of the spherical model.

Learn  more about exponential function here:https://brainly.com/question/2456547

#SPJ2

Taxi A charges a fee of $3.50, plus $1.75 per mile. Taxi B charges a few of $1.25, plus $2.00 per mile. At what distance would the taxi cost the same?

Answers

Taxi A
1mile £3.50+£1.75=£5.25
Taxi B
1mile £1.25+£2.00=£3.25

Taxi A
2miles £3.50+£3.50=£7.00
Taxi B
2miles £1.25+£4.00=£5.25

Taxi A
3miles £3.50+£5.25=£8.75
Taxi B
3miles £1.25+£6.00=£7.25

Taxi A
4miles £3.50+£7.00=£10.50
Taxi B
4miles £1.25+£8.00=£9.25

Taxi A
5miles £3.50+£8.75=£12.25
Taxi B
5miles £1.25+£10.00=£11.25

Taxi A
6miles £3.50+£10.50=£14.00
Taxi B
6miles £1.25+£12.00=£13.25

Taxi A
7miles £3.50+£12.25=£15.75
Taxi B
7miles £1.25+£14.00=£15.25

Taxi A
8miles £3.50+£14.00=£17.50
Taxi B
8miles £1.25+£16.00=£17.25

Taxi A
9miles £3.50+£15.75=£19.25 (the same)
Taxi B
9miles £1.25+£18.00=£19.25 (the same)

^^^
They would have to drive 9 miles for the taxi to cost the same.

Hope this helped, this is the longest way to work it out but also the simplest.

Which inverse of the difference of 1/3 - 1/4?

Answers

Answer:

0/1

Step-by-step explanation:

PLEASE HELPP MEEE ASAPPPPP

Answers

Answer:

D) 24,32,40

Step-by-step explanation:

Sense this is a right triangle, I will use the Pythagorean theorem to solve this problem. Use the following formula: a^2 + b^2 = c^2. 40 would be c. 40 x 40 = 1600. 32 x 32 = 1024.  24 x 24 = 576. Since 576 + 1,024 equals 1600, this means the answer would be D. Remember, that you can only use the Pythagorean Theorem method on right triangles.

1,600 = 576 + 1024.

Note:

Pls notify me if my answer is incorrect, for the other users that will see this message.

-kiniwih426

Find the mean, median, mode, range.

Answers

Answer:

1. mean: 5

2. median: 6

3. mode: 80% or 8

4. range: 7

Step-by-step explanation:

Answer:

mean = 5

median = 6

mode = 80%(aka 8)

range = 7

Step-by-step explanation:

Solve for x
Show work

Answers

Answer:

10x is equal to 100 so 100+6=106

then

Step-by-step explanation:

4x is equal to 8 so 4+18=22

so it's 128

Darius has a 6 month loan for $500. He must pay 5.6% annual interest on the loan. Using the formula for simple interest, I=Prt, where I is interest owed, P is the amount borrowed, r is the rate as a decimal, and t is time in years, find the amount of interest owed by Darius after 6 months

A. $16.80
B. $140
C. $168
D. $14

Answers

This will be B hope this helps

Answer:

14

Step-by-step explanation:

- - - - - - - - - - - - - - - - - - - - - - - - -

Answers

Answer:

x = ±3 sqrt(2)

Step-by-step explanation:

x^2 = 18

Take the square root of each side

sqrt(x^2) = sqrt(18)

x = ± sqrt(18)

x =±sqrt(9*2)

x = ±sqrt(9) sqrt(2)

x = ±3 sqrt(2)

Which of the following is an equivalent trig ratio for tan 28

Cos 62

1/ tan 62

1/ tan152

Cos 28

Answers

Answer:

B

Step-by-step explanation:

tan28=tan (90-62)=cot 62=1/tan 62

Adam is refinishing his basement. 30 ft by 50 ft are the dimensions of Adam's rectangular basement. How much tile would he need to cover the floor? If the height of the basement is 6 ft, how much wall area is there to paint?

Answers

Answer:

Area of four walls is 3960 square feet.

Step-by-step explanation:

length, L = 30 ft

Width, W = 50 ft

height, H = 6 ft

The area of walls is

= 2 (L W + W H + H L)

= 2 (30 x 50 + 50 x 6 + 6 x 30)

= 2 (1500 + 300 + 180)

= 3960 square feet

Answer/Step-by-step explanation:

How much tile would he need to cover the floor?

Answer: 1500 Ft^2

If the height of the basement is 6 ft, how much wall area is there to paint?

Answer: 960 Ft^2


Sophia drops her pencil in a cup and
notices that it only fits diagonally.
The pencil is 10 centimeters long and the
cup is 8 centimeters tall.
What is the diameter of the cup?

Answers

Answer:

5cm

Step-by-step explanation:

Since the pencil would only fit diagonally, the diameter would have to be half its length. The pencil is 10 centimeters long and 5 centimeters would be half of it.

Note:

Pls notify me if my answer is incorrect for the other users that will see this message. Thank you.

-kiniwih426

Find a round to the nearest tenth 12 22 75 x x=?

Answers

Answer:

By law of Sines[tex]\frac{Sin75^o}{22} =\frac{Sinx}{12}[/tex][tex]\frac{Sin75}{22}(12)=\frac{Sinx}{12} (12)[/tex][tex]0.5268=Sinx[/tex][tex]Sin^(0.5268)=x[/tex][tex]x=31.789[/tex][tex]x=31.79^o[/tex]-----------------------hope it helps..have a great day!!

Which correctly uses bar notation to represent the repeating decimal for
fi
O 0.54
O 05454
O 054
O 0.545

Answers

0.45 I took the test

The volume of this cone is 83.7 cubic meters. Find the DIAMETER. SHOW ALL WORK

Answers

There for the diameter is 2(4.8 )= 9.6 ft

Given: Volume of the cone is 83.7 m³

We know that:

[tex]\bigstar \ \ \boxed{\sf{\textsf{Volume of cone is given by} : \dfrac{\pi r^2h}{3}}}[/tex]

where r is the radius of the cone and h is the height of the cone

Given: Height of the cone = 5m

Substituting the values in the formula, we get:

[tex]\sf{\implies \dfrac{\pi r^2(5)}{3} = 83.7}[/tex]

[tex]\sf{\implies \pi r^2 = 83.7 \times \dfrac{3}{5}}[/tex]

[tex]\sf{\implies \pi r^2 = 83.7 \times 0.6}[/tex]

[tex]\sf{\implies \pi r^2 = 50.22}[/tex]

[tex]\sf{\implies r^2 = \dfrac{50.22}{\pi}}[/tex]

[tex]\sf{\implies r^2 = 16}[/tex]

[tex]\sf{\implies r = 4}[/tex]

We know that : Diameter is two times the radius

⇒  Diameter of the Cone is 8 meters

the dishes have been cleaned by us into active voice​

Answers

Answer:

the dishes sparkled after we washed them.

Step-by-step explanation:

The dishes are clean once you wash them duh

Pls help me with this

Answers

Answer:

5.85

Step-by-step explanation:

100%-> 6.50

100%-10% = 90 %

90%-> 6.50/100 x 90

           =5.85

If a square table can fit 8 people and a round table can fit 6 people, the equation 150= 8x + 6y , if Y =9 what is the value of x ? The committee needs how many square tables?

Answers

Answer:

The committee needs 12 square tables!

Step-by-step explanation:

[tex]8x+6y=150[/tex]

[tex]y=9[/tex]

[tex]8x+6(9)=150[/tex]

[tex]8x+54=150[/tex]

      [tex]-54[/tex]     [tex]-54[/tex]

[tex]8x=96[/tex]

[tex]/8[/tex]      [tex]/8[/tex]

[tex]x=12[/tex]

Hope this helps!

The circumference of a circle is 8.34cm. Find the length of the diameter. Give your answer rounded to 2 DP.

Answers

Answer:

Diameter, D = 2.65 cm

Step-by-step explanation:

Given the following data;

Circumference of circle = 8.34 cm

To find the length of its diameter;

Mathematically, circumference of a circle is equal to;

C = πD

Where;

C is the circumference of a circle.D is the diameter of a circle.Pie, π = 3.142

Substituting into the formula, we have;

8.34 = 3.142 * D

[tex] D = \frac {8.34}{3.142} [/tex]

Diameter, D = 2.65 cm

A function g is described below:
· g(2) = 2 ( 23 – 3) = 5
• domain is all real numbers greater than 0
The range of g is all real numbers greater than or equal to

Answers

Answer:

A≈1075.21

d Diameter

37

d

r

r

r

d

d

C

A

Using the formulas

A=

π

r

2

d=

2

r

Solving forA

A=

1

4

π

d

2

=

1

4

π

37

2

1075.21009

Step-by-step explanation:

Frank earned f dollars mowing lawns. Bryce's earnings, b, were $15.25 less than 3.5 times as
much as Frank's earnings. Which of the following equations best represents this situation?
A. b = 3.5f + 15.25
B. f= 3.5b - 15.25
C. b = 3.5f-15.25
D. f = 3.5b + 15.25

Answers

it’s D. f = 3.5b + 15.25

BRAINLIEST IF CORRECT You choose a movie at random from a list containing 8 comedy movies, 5 science fiction movies, and 7 adventure movies. What is the theoretical probability that the movie is not a comedy?

Select all that apply.

0.60

50%

252 fifths

353 fifths

60%

Answers

Answer:

0.60 & 60%

Step-by-step explanation:

Please answer this fasttt

Answers

2*2*3
Ra 2*2*3*3 huncha
Hope it helps : )

The sum of a number and its square is 12 . Find the number

Answers

Answer:

Two solutions

3

-4

Step-by-step explanation:

3 + 3²

3 + 9 = 12

-------------------

-4  + (-4)²

-4 + 16 = 12

The isosceles triangle and the rectangle have equal perimeters. The triangle has x+5 on the left side. On its base it has x. The rectangle has x-1 on the left side. On the bottom, it has x+4.

Answers

Answer:

The value of x is 4.

Step-by-step explanation:

The perimeter of the triangle is the sum of the lengths of all its sides.

The perimeter of the rectangle is twice the sum of the length and the width.

base of triangle = x

two equal sides = x + 5

length of rectangle = x - 1

width of rectangle = x + 4

According to the question,

Perimeter of triangle = perimeter of rectangle

x + x + 5 + x + 5 = 2 ( x - 1 + x + 4)

3 x + 10 = 2 (2 x + 3 )

3 x + 10 = 4 x + 6

4 x - 3 x = 10 - 6

x = 4

HELP ME PLEASE DUE SOON!!!!!! WILL MARK BRAINLIEST

Answers

P’Q 4
A’B’ 3
M’N 6 good luck on the rest of ur test :>

A swimmer breaks a world record by 0.07 seconds. The old record was 49.51 seconds. What is the swimmer's new world record?
a. 49.58 seconds
b.49.44 seconds
c.48.81 seconds
d.48.58 seconds

Answers

Answer:

B

Step-by-step explanation:

Simply subtract 0.07 from 49.51 too get 49.44.

Hope that this helps!

Other Questions
Learning Activity DetailsAmerican Red CrossTraining Services00Red BocAfter the resuscitation event, the BLS team conducted a debriefingsession to ensure continuous CPR quality improvement. Whichobservation or data point suggests a need for improvement?Select the correct answer to this question.Joanne used closed-loop communication to confirm she received and understoodValerie's directionThe feedback device indicated that Kirron's compression depth was between 2.1and 2.3 inches deep.Valerie observed signs of compressor fatigue and directed the team to switchpositions.The chest compression fraction (CCF) was 58%. Could anyone help me please? Which of the following is NOT a reason for theorigins of government?A. Defend private propertyB. Collect tax revenuesC. Protect people from conflictD. Provide law and order If GH = HI And FI =10 what is GI quistion in picture only for WXI |- 3x| >= - 7 CAN SOMEONE PLEASE GIVE ME THE SOLUTION SET TO THIS INEQUALITY( WORTH 100 POINTS) what is the length of the missing leg? Help me!! Please help me find the answer!! 6x - y= 16 and 3x + 2y + -12 Answer. In 150 to 175 words, analyzes and give your opinion of the following statement. Antagonism arose among the conquered nomadic tribes of the Xiongnu. A news report suggested that an adult should drink a minimum of 4 pints of water per day.Based on this report, determine the minimum amount of water an adult should drink,in fluid ounces, per week. PLS ANSWER NOWWWWWWWWWWWWWWWWW Explain what happens to an animal cell when it removed from an organism and placed into distilled water is the government doing enough to ensure that gender based violence protected explain Can someone help me please POR QU ES TAN IMPORTANTE PARA LAS ESPECIES EL OLFATO? Imagine you are reading a story about a character named Lula. How could the headline best be revised to make it objective?How could the first sentence of the article be revised to improve objectivity?How could the reporter balance Senator Sauds comments to make the article more objective? An unstretched ideal spring hangs vertically from a fixed support. A 0.4 kg object is then attached to the lower end of the spring. The object is pulled down to a distance of 0.35 m below the unstretched position and released from rest at time t= 0. A graph of the subsequent vertical position y of the lower end of the spring as a function of t is given above, where y= 0 when the spring was initially unstretched. At which time is the upward velocity of the object the greatest? How does the rest of the community react to what Moishe the Beadle tells them about their future?